You are on page 1of 26

Chapter 43: Assessment of Digestive and Gastrointestinal 6.

A patient has come to the outpatient radiology department


Function for diagnostic testing. Which of the following diagnostic
procedures will allow the care team to evaluate and remove
1. A nurse is caring for a patient who is scheduled for a
polyps?
colonoscopy and whose bowel preparation will include
A) Colonoscopy
polyethylene glycol electrolyte lavage prior to the procedure.
B) Barium enema
The presence of what health problem would contraindicate
C) ERCP
the use of this form of bowel preparation?
D) Upper gastrointestinal fibroscopy
A) Inflammatory bowel disease
B) Intestinal polyps
7. A nurse is caring for a patient with recurrent hematemesis
C) Diverticulitis
who is scheduled for upper gastrointestinal
D) Colon cancer
fibroscopy (UGF). How should the nurse in the radiology
department prepare this patient?
2. A nurse is promoting increased protein intake to enhance a
A) Insert a nasogastric tube.
patients wound healing. The nurse knows
B) Administer a micro Fleet enema at least 3 hours before the
that enzymes are essential in the digestion of nutrients such
procedure.
as protein. What is the enzyme that initiates
C) Have the patient lie in a supine position for the procedure.
the digestion of protein?
D) Apply local anesthetic to the back of the patients throat.
A) Pepsin
B) Intrinsic factor
8. The nurse is providing health education to a patient
C) Lipase
scheduled for a colonoscopy. The nurse should explain that
D) Amylase
she will be placed in what position during this diagnostic test?
A) In a knee-chest position (lithotomy position)
3. A patient has been brought to the emergency department
B) Lying prone with legs drawn toward the chest
with abdominal pain and is subsequently
C) Lying on the left side with legs drawn toward the chest
diagnosed with appendicitis. The patient is scheduled for an
D) In a prone position with two pillows elevating the buttocks
appendectomy but questions the nurse about how his health
will be affected by the absence of an appendix. How should
9. A patient has sought care because of recent dark-colored
the nurse best respond?
stools. As a result, a fecal occult blood test has
A) Your appendix doesnt play a major role, so you wont notice
been ordered. The nurse should instruct the patient to avoid
any difference after you recovery from surgery.
which of the following prior to collecting a
B) The surgeon will encourage you to limit your fat intake for a
stool sample?
few weeks after the surgery, but your body will then begin to
A) NSAIDs
compensate.
B) Acetaminophen
C) Your body will absorb slightly fewer nutrients from the food
C) OTC vitamin D supplements
you eat, but you wont be aware of this.
D) Fiber supplements
D) Your large intestine will adapt over time to the absence of
your appendix.
10. The nurse is preparing to perform a patients abdominal
assessment. What examination sequence should the nurse
4. A patient asks the nursing assistant for a bedpan. When the
follow?
patient is finished, the nursing assistant notifies the nurse that
A) Inspection, auscultation, percussion, and palpation
the patient has bright red streaking of blood in the stool.
B) Inspection, palpation, auscultation, and percussion
What is this most likely a result of?
C) Inspection, percussion, palpation, and auscultation
A) Diet high in red meat
D) Inspection, palpation, percussion, and auscultation
B) Upper GI bleed
C) Hemorrhoids
11. A patient who has been experiencing changes in his bowel
D) Use of iron supplements
function is scheduled for a barium enema.
5. An adult patient is scheduled for an upper GI series that will What instruction should the nurse provide for postprocedure
use a barium swallow. What teaching should the nurse include recovery?
when the patient has completed the test? A) Remain NPO for 6 hours postprocedure.
A) Stool will be yellow for the first 24 hours postprocedure. B) Administer a Fleet enema to cleanse the bowel of the
B) The barium may cause diarrhea for the next 24 hours. barium.
C) Fluids must be increased to facilitate the evacuation of the C) Increase fluid intake to evacuate the barium.
stool. D) Avoid dairy products for 24 hours postprocedure.
D) Slight anal bleeding may be noted as the barium is passed.
12. A nurse is caring for a newly admitted patient with a
suspected GI bleed. The nurse assesses the patients stool after
1
a bowel movement and notes it to be a tarry-black color. This B) Chronic jaundice in the absence of liver disease
finding is suggestive of bleeding from what location? C) The presence of fat in the patients stool
A) Sigmoid colon D) Persistently low hemoglobin and hematocrit
B) Upper GI tract
C) Large intestine 19. A patient with a recent history of intermittent bleeding is
D) Anus or rectum undergoing capsule endoscopy to determine the source of the
bleeding. When explaining this diagnostic test to the patient,
13. A nursing student has auscultated a patients abdomen and what advantage should the nurse describe?
noted one or two bowel sounds in a 2-minute period of time. A) The test allows visualization of the entire peritoneal cavity.
How would you tell the student to document the patients B) The test allows for painless biopsy collection.
bowel sounds? C) The test does not require fasting.
A) Normal D) The test is noninvasive.
B) Hypoactive
C) Hyperactive 20. A nurse is caring for a patient admitted with a suspected
D) Paralytic ileus malabsorption disorder. The nurse knows that
one of the accessory organs of the digestive system is the
14. An advanced practice nurse is assessing the size and pancreas. What digestive enzymes does the pancreas secrete?
density of a patients abdominal organs. If the results of Select all that apply.
palpation are unclear to the nurse, what assessment A) Pepsin
technique should be implemented? B) Lipase
A) Percussion C) Amylase
B) Auscultation D) Trypsin
C) Inspection E) Ptyalin
D) Rectal examination
21. The nurse is caring for a patient with a duodenal ulcer and
15. A nurse is caring for a patient with biliary colic and is is relating the patients symptoms to the physiologic functions
aware that the patient may experience referred of the small intestine. What do these functions include? Select
abdominal pain. Where would the nurse most likely expect all that apply.
this patient to experience referred pain? A) Secretion of hydrochloric acid (HCl)
A) Midline near the umbilicus B) Reabsorption of water
B) Below the right nipple C)Secretion of mucus
C) Left groin area D) Absorption of nutrients
D) Right lower abdominal quadrant E) Movement of nutrients into the bloodstream
.
16. An inpatient has returned to the medical unit after a 22. A nurse is performing an abdominal assessment of an
barium enema. When assessing the patients older adult patient. When collecting and analyzing data, the
subsequent bowel patterns and stools, what finding should nurse should be cognizant of what age-related change in
the nurse report to the physician? gastrointestinal structure and function?
A) Large, wide stools A) Increased gastric motility
B) Milky white stools B) Decreased gastric pH
C) Three stools during an 8-hour period of time C) Increased gag reflex
D) Streaks of blood present in the stool D) Decreased mucus secretion

17. A nurse in a stroke rehabilitation facility recognizes that 23. The nurse educator is reviewing the blood supply of the GI
the brain regulates swallowing. Damage to what tract with a group of medical nurses. The nurse is explaining
area of the brain will most affect the patients ability to the fact that the veins that return blood from the digestive
swallow? organs and the spleen form the portal venous system. What
A) Temporal lobe large veins will the nurse list when describing this system?
B) Medulla oblongata Select all that apply.
C) Cerebellum A) Splenic vein
D) Pons B) Inferior mesenteric vein
C) Gastric vein
18. A patient is being assessed for a suspected deficit in D) Inferior vena cava
intrinsic factor synthesis. What diagnostic or assessment 24. The physiology instructor is discussing the GI system with
finding is the most likely rationale for this examination of the pre-nursing class. What should the instructor describe as a
intrinsic factor production? major function of the GI tract?
A) Muscle wasting A) The breakdown of food particles into cell form for digestion
2
B) The maintenance of fluid and acid-base balance What would indicate that these medications have had the
C) The absorption into the bloodstream of nutrient molecules desired therapeutic effect?
produced by digestion A) The patients BUN and creatinine levels are within reference
D) The control of absorption and elimination of electrolytes range following the CT.
B) The CT yields high-quality images.
25. A nurse is providing preprocedure education for a patient C) The patients electrolytes are stable in the 48 hours
who will undergo a lower GI tract study the following the CT.
following week. What should the nurse teach the patient D) The patients intake and output are in balance on the day
about bowel preparation? after the CT.
A) You'll need to fast for at least 18 hours prior to your test.
B) Starting today, take over-the-counter stool softeners twice 30. A medical patients CA 19-9 levels have become available
daily. and they are significantly elevated. How
C) You'll need to have enemas the day before the test. should the nurse best interpret this diagnostic finding?
D) For 24 hours before the test, insert a glycerin suppository A) The patient may have cancer, but other GI disease must be
every 4 hours. ruled out.
B) The patient most likely has early-stage colorectal cancer.
26. A patient presents at the walk-in clinic complaining of C) The patient has a genetic predisposition to gastric cancer.
recurrent sharp stomach pain that is relieved by eating. The D) The patient has cancer, but the site is unknown.
nurse suspects that the patient may have an ulcer. How would
the nurse explain the formation and role of acid in the 31. A patient has come to the clinic complaining of blood in his
stomach to the patient? stool. A FOBT test is performed but is negative. Based on the
A) Hydrochloric acid is secreted by glands in the stomach in patients history, the physician suggests a colonoscopy, but the
response to the actual or anticipated presence of food. patient refuses, citing a strong aversion to the invasive nature
B) As digestion occurs in the stomach, the stomach combines of the test. What other test might the physician order to
free hydrogen ions from the food to check for blood in the stool?
form acid. A) A laparoscopic intestinal mucosa biopsy
C) The body requires an acidic environment in order to B) A quantitative fecal immunochemical test
synthesize pancreatic digestive enzymes; the stomach C) Computed tomography (CT)
provides this environment. D) Magnetic resonance imagery (MRI)
D) The acidic environment in the stomach exists to buffer the
highly alkaline environment in the esophagus. 32. A nurse is assessing the abdomen of a patient just
admitted to the unit with a suspected GI disease.
27. Results of a patients preliminary assessment prompted an Inspection reveals several diverse lesions on the patients
examination of the patients carcinoembryonic antigen (CEA) abdomen. How should the nurse best interpret this
levels, which have come back positive. What is the nurses assessment finding?
most appropriate response to this finding? A) Abdominal lesions are usually due to age-related skin
A) Perform a focused abdominal assessment. changes.
B) Prepare to meet the patients psychosocial needs. B) Integumentary diseases often cause GI disorders.
C) Liaise with the nurse practitioner to perform an anorectal C) GI diseases often produce skin changes.
examination. D) The patient needs to be assessed for self-harm.
D) Encourage the patient to adhere to recommended
screening protocols.

28. A clinic patient has described recent dark-colored


stools;the nurse recognizes the need for fecal occult
blood testing (FOBT). What aspect of the patients current
health status would contraindicate FOBT?
A) Gastroesophageal reflux disease (GERD)
B) Peptic ulcers
C) Hemorrhoids
D) Recurrent nausea and vomiting

29. A patient will be undergoing abdominal computed 33.Probably the most widely used in-office or at-home occult
tomography (CT) with contrast. The nurse has blood test is the Hemoccult II. The patient has come to the
administered IV sodium bicarbonate and oral acetylcysteine clinic because he thinks there is blood in his stool. When you
(Mucomyst) before the study as ordered. reviewed his medications, you noted he is on antihypertensive
drugs and NSAIDs for early arthritic pain. You are sending the
3
patient home with the supplies necessary to perform 2 D) Inform the primary care provider of this finding.
hemoccult tests on his stool and mail the samples back to the
clinic. What instruction would you give this patient? 39. A patient has been scheduled for a urea breath test in one
A) Take all your medications as usual. months time. What nursing diagnosis most likely prompted
B) Take all your medications except the antihypertensive this diagnostic test?
medications. A) Impaired Dentition Related to Gingivitis
C) Dont eat highly acidic foods 72 hours before you start the B) Risk For Impaired Skin Integrity Related to Peptic Ulcers
test. C) Imbalanced Nutrition: Less Than Body Requirements
D) Avoid vitamin C for 72 hours before you start the test. Related to Enzyme Deficiency
D) Diarrhea Related to Clostridium Difficile Infection
34. A patients sigmoidoscopy has been successfully completed 40. A female patient has presented to the emergency
and the patient is preparing to return home. department with right upper quadrant pain; the
Which of the following teaching points should the nurse physician has ordered abdominal ultrasound to rule out
include in the patients discharge education? cholecystitis (gallbladder infection). The patient
A) The patient should drink at least 2 liters of fluid in the next expresses concern to the nurse about the safety of this
12 hours. diagnostic procedure. How should the nurse best respond?
B) The patient can resume a normal routine immediately. A) Abdominal ultrasound is very safe, but it cant be performed
C) The patient should expect fecal urgency for several hours. if youre pregnant.
D) The patient can expect some scant rectal bleeding. B) Abdominal ultrasound poses no known safety risks of any
kind.
35. A nurse is caring for an 83-year-old patient who is being C) Current guidelines state that a person can have up to 3
assessed for recurrent and intractable nausea. ultrasounds per year.
What age-related change to the GI system may be a D) Current guidelines state that a person can have up to 6
contributor to the patients health complaint? ultrasounds per year.
A) Stomach emptying takes place more slowly.
B) The villi and epithelium of the small intestine become Chapter 44: Digestive and Gastrointestinal Treatment
thinner.
Modalities
C) The esophageal sphincter becomes incompetent.
D) Saliva production decreases. 1. A nurse is preparing to place a patients ordered nasogastric
tube. How should the nurse best determine
36. A patient has been experiencing significant psychosocial the correct length of the nasogastric tube?
stress in recent weeks. The nurse is aware of A) Place distal tip to nose, then ear tip and end of xiphoid
the hormonal effects of stress, including norepinephrine process.
release. Release of this substance would have what effect on B) Instruct the patient to lie prone and measure tip of nose to
the patients gastrointestinal function? umbilical area.
Select all that apply. C) Insert the tube into the patients nose until secretions can
A) Decreased motility be aspirated.
B) Increased sphincter tone D) Obtain an order from the physician for the length of tube
C) Increased enzyme release to insert.
D) Inhibition of secretions
2. A patient is concerned about leakage of gastric contents out
E) Increased peristalsis
of the gastric sump tube the nurse has just inserted. What
would the nurse do to prevent reflux gastric contents from
37. A patient with cystic fibrosis takes pancreatic enzyme
coming through the blue vent of a gastric sump tube?
replacements on a regular basis. The patients intake of trypsin
A) Prime the tubing with 20 mL of normal saline.
facilitates what aspect of GI function?
B) Keep the vent lumen above the patients waist.
A) Vitamin D synthesis
C) Maintain the patient in a high Fowlers position.
B) Digestion of fats
D) Have the patient pin the tube to the thigh.
C) Maintenance of peristalsis
D) Digestion of proteins
3. A patient receiving tube feedings is experiencing diarrhea.
The nurse and the physician suspect that the
38. The nurse is caring for a patient who has a diagnosis of
patient is experiencing dumping syndrome. What intervention
AIDS. Inspection of the patients mouth reveals the new
is most appropriate?
presence of white lesions on the patients oral mucosa. What
A) Stop the tube feed and aspirate stomach contents.
is the nurses most appropriate response?
B) Increase the hourly feed rate so it finishes earlier.
A) Encourage the patient to gargle with salt water twice daily.
C) Dilute the concentration of the feeding solution.
B) Attempt to remove the lesions with a tongue depressor.
D) Administer fluid replacement by IV.
C) Make a referral to the units dietitian.
4
the infusion site after running the emulsion through a filter.
4. A nurse is admitting a patient to the postsurgical unit D) The intravenous fat emulsions can be piggy-backed into any
following a gastrostomy. When planning assessments, the existing IV solution that is infusing.
nurse should be aware of what potential postoperative
complication of a gastrostomy? 9. A nurse is participating in a patients care conference and
A) Premature removal of the G tube the team is deciding between parenteral nutrition (PN) and a
B) Bowel perforation total nutritional admixture (TNA). What advantages are
C) Constipation associated with providing TNA rather than PN?
D) Development of peptic ulcer disease (PUD) A) TNA can be mixed by a certified registered nurse.
B) TNA can be administered over 8 hours, while PN requires
5. A nursing educator is reviewing the care of patients with 24-hour administration.
feeding tubes and endotracheal tubes (ET). The C) TNA is less costly than PN.
educator has emphasized the need to check for tube D) TNA does not require the use of a micron filter.
placement in the stomach as well as residual volume. What is
the main purpose of this nursing action? 10. A nurse is initiating parenteral nutrition (PN) to a
A) Prevent gastric ulcers postoperative patient who has developed complications. The
B) Prevent aspiration nurse should initiate therapy by performing which of the
C) Prevent abdominal distention following actions?
D) Prevent diarrhea A) Starting with a rapid infusion rate to meet the patients
nutritional needs as quickly as possible
6.The nurse is administering total parenteral nutrition (TPN) B) Initiating the infusion slowly and monitoring the patients
to a client who underwent surgery for gastric fluid and glucose tolerance
cancer. Which of the nurses assessments most directly C) Changing the rate of administration every 2 hours based on
addresses a major complication of TPN? serum electrolyte values
A) Checking the patients capillary blood glucose levels D) Increasing the rate of infusion at mealtimes to mimic the
regularly circadian rhythm of the body
B) Having the patient frequently rate his or her hunger on a
10-point scale 11. A patients physician has determined that for the next 3 to
C) Measuring the patients heart rhythm at least every 6 hours 4 weeks the patient will require parenteral nutrition (PN). The
D) Monitoring the patients level of consciousness each shift nurse should anticipate the placement of what type of venous
access device?
7. A critical care nurse is caring for a patient diagnosed with A) Peripheral catheter
acute pancreatitis. The nurse knows that the B) Nontunneled central catheter
indications for starting parenteral nutrition (PN) for this C) Implantable port
patient are what? D) Tunneled central catheter
A) 5% deficit in body weight compared to preillness weight
and increased caloric need 12. A nurse is caring for a patient who has an order to
B) Calorie deficit and muscle wasting combined with low discontinue the administration of parenteral nutrition.
electrolyte levels What should the nurse do to prevent the occurrence of
C) Inability to take in adequate oral food or fluids within 7 rebound hypoglycemia in the patient?
days A) Administer an isotonic dextrose solution for 1 to 2 hours
D) Significant risk of aspiration coupled with decreased level after discontinuing the PN.
of consciousness B) Administer a hypertonic dextrose solution for 1 to 2 hours
after discontinuing the PN.
8. A nurse is preparing to administer a patients intravenous C) Administer 3 ampules of dextrose 50% immediately prior to
fat emulsion simultaneously with parenteral discontinuing the PN.
nutrition (PN). Which of the following principles should guide D) Administer 3 ampules of dextrose 50% 1 hour after
the nurses action? discontinuing the PN.
A) Intravenous fat emulsions may be infused simultaneously 13. A nurse is caring for a patient with a subclavian central
with PN through a Y-connector close to line who is receiving parenteral nutrition (PN). In preparing a
the infusion site and should not be filtered. care plan for this patient, what nursing diagnosis should the
B) The nurse should prepare for placement of another nurse prioritize?
intravenous line, as intravenous fat emulsions A) Risk for Activity Intolerance Related to the Presence of a
may not be infused simultaneously through the line used for Subclavian Catheter
PN. B) Risk for Infection Related to the Presence of a Subclavian
C) Intravenous fat emulsions may be infused simultaneously Catheter
with PN through a Y-connector close to
5
C) Risk for Functional Urinary Incontinence Related to the B) Assess the color and pH of aspirate.
Presence of a Subclavian Catheter C) Locate the marking made after the initial x-ray confirming
D) Risk for Sleep Deprivation Related to the presence of a placement.
Subclavian Catheter D) Use a combination of at least two accepted methods for
confirming placement.
14. A patients health decline necessitates the use of total
parenteral nutrition. The patient has questioned the 19. The nurse is assessing placement of a nasogastric tube
need for insertion of a central venous catheter, expressing a that the patient has had in place for 2 days. The tube is
preference for a normal IV. The nurse should know that draining green aspirate. What is the nurses most appropriate
peripheral administration of high-concentration PN formulas action?
is contraindicated because of the risk for what complication? A) Inform the physician that the tube may be in the patients
A) Chemical phlebitis pleural space.
B) Withdraw the tube 2 to 4 cm.
15. A nurse is providing care for a patient with a diagnosis of C) Leave the tube in its present position.
late-stage Alzheimers disease. The patient has just returned to D) Advance the tube up to 8 cm.
the medical unit to begin supplemental feedings through an
NG tube. Which of the nurses assessments addresses this 20. A patients new onset of dysphagia has required insertion
patients most significant potential complication of feeding? of an NG tube for feeding; the nurse has
A) Frequent assessment of the patients abdominal girth modified the patients care plan accordingly. What
B) Assessment for hemorrhage from the nasal insertion site intervention should the nurse include in the patients
C) Frequent lung auscultation plan of care?
D) Vigilant monitoring of the frequency and character of A) Confirm placement of the tube prior to each medication
bowel movements administration.
B) Have the patient sip cool water to stimulate saliva
16. The management of the patients gastrostomy is an production.
assessment priority for the home care nurse. What C) Keep the patient in a low Fowlers position when at rest.
statement would indicate that the patient is managing the D) Connect the tube to continuous wall suction when not in
tube correctly? use.
A) I clean my stoma twice a day with alcohol.
B) The only time I flush my tube is when Im putting in 21. A patient has been brought to the emergency department
medications. by EMS after telling a family member that he
C) I flush my tube with water before and after each of my deliberately took an overdose of NSAIDs a few minutes
medications. earlier. If lavage is ordered, the nurse should prepare to assist
D) I try to stay still most of the time to avoid dislodging my with the insertion of what type of tube?
tube. A) Nasogastric tube
B) Levin tube
17. A nurse is caring for a patient with a nasogastric tube for C) Gastric sump
feeding. During shift assessment, the nurse D) Orogastric tube
auscultates a new onset of bilateral lung crackles and notes a
respiratory rate of 30 breaths per minute.
The patients oxygen saturation is 89% by pulse oximetry. After
ensuring the patients immediate safety,
what is the nurses most appropriate action?
A) Perform chest physiotherapy.
B) Reduce the height of the patients bed and remove the NG
tube.
C) Liaise with the dietitian to obtain a feeding solution with
lower osmolarity. 22. A patients NG tube has become clogged after the nurse
D) Report possible signs of aspiration pneumonia to the instilled a medication that was insufficiently crushed. The
primary care provider. nurse has attempted to aspirate with a large-bore syringe,
with no success. What should the nurse do next?
18. A nurse is creating a care plan for a patient with a A) Withdraw the NG tube 3 to 5 cm and reattempt aspiration.
nasogastric tube. How should the nurse direct other B) Attach a syringe filled with warm water and attempt an in-
members of the care team to check correct placement of the and-out motion of instilling and aspirating.
tube? C) Withdraw the NG tube slightly and attempt to dislodge by
A) Auscultate the patients abdomen after injecting air through flicking the tube with the fingers.
the tube.
6
D) Remove the NG tube promptly and obtain an order for 28. A patient with dysphagia is scheduled for PEG tube
reinsertion from the primary care provider. insertion and asks the nurse how the tube will stay
in place. What is the nurses best response?
23. A nurse has obtained an order to remove a patients NG A) Adhesive holds a flange in place against the abdominal skin.
tube and has prepared the patient accordingly. After flushing B) A stitch holds the tube in place externally.
the tube and removing the nasal tape, the nurse attempts C) The tube is stitched to the abdominal skin externally and
removal but is met with resistance. Because the nurse is the stomach wall internally.
unable to overcome this resistance, what is the most D) An internal retention disc secures the tube against the
appropriate action? stomach wall.
A) Gently twist the tube before pulling.
B) Instill a digestive enzyme solution and reattempt removal in 29. A patient is postoperative day 1 following gastrostomy.
10 to 15 minutes. The nurse is planning interventions to address the nursing
C) Flush the tube with hot tap water and reattempt removal. diagnosis of Risk for Infection Related to Presence of Wound
D) Report this finding to the patients primary care provider. and Tube. What intervention is
most appropriate?
24. A nurse is writing a care plan for a patient with a A) Administer antibiotics via the tube as ordered.
nasogastric tube in place for gastric decompression. B) Wash the area around the tube with soap and water daily.
What risk nursing diagnosis is the most appropriate C) Cleanse the skin within 2 cm of the insertion site with
component of the care plan? hydrogen peroxide once per shift.
A) Risk for Excess Fluid Volume Related to Enteral Feedings D) Irrigate the skin surrounding the insertion site with normal
B) Risk for Impaired Skin Integrity Related to the Presence of saline before each use.
NG Tube
C) Risk for Unstable Blood Glucose Related to Enteral Feedings 30. The nurse is preparing to insert a patients ordered NG
tube. What factor should the nurse recognize as a
25. A patients enteral feedings have been determined to be risk for incorrect placement?
too concentrated based on the patients development of A) The patient is obese and has a short neck.
dumping syndrome. What physiologic phenomenon caused B) The patient is agitated.
this patients complication of enteral feeding? C) The patient has a history of gastroesophageal reflux disease
A) Increased gastric secretion of HCl and gastrin because of (GERD).
high osmolality of feeds D) The patient is being treated for pneumonia.
B) Entry of large amounts of water into the small intestine
because of osmotic pressure 31. Prior to a patients scheduled jejunostomy, the nurse is
C) Mucosal irritation of the stomach and small intestine by the performing the preoperative assessment. What goal should
high concentration of the feed the nurse prioritize during the preoperative assessment?
D) Acidbase imbalance resulting from the high volume of A) Determining the patients nutritional needs
solutes in the feed B) Determining that the patient fully understands the
postoperative care required
26. A nurse is creating a care plan for a patient who is C) Determining the patients ability to understand and
receiving parenteral nutrition. The patients care plan cooperate with the procedure
should include nursing actions relevant to what potential D) Determining the patients ability to cope with an altered
complications? Select all that apply. body image
A) Dumping syndrome
B) Clotted or displaced catheter
C) Pneumothorax
D) Hyperglycemia
E) Line sepsis 32. You are caring for a patient who was admitted to have a
27. A nurse is caring for a patient who has a gastrointestinal low-profile gastrostomy device (LPGD) placed. How soon after
tube in place. Which of the following are the original gastrostomy tube placement can an LPGD be
indications for gastrointestinal intubation? Select all that placed?
apply. A) 2 weeks
A) To remove gas from the stomach B) 4 to 6 weeks
B) To administer clotting factors to treat a GI bleed C) 2 to 3 months
C) To remove toxins from the stomach
D) To open sphincters that are closed 33. A nurse is caring for a patient who is receiving parenteral
E) To diagnose GI motility disorders nutrition. When writing this patients plan of
care, which of the following nursing diagnoses should be
included?
7
A) Risk for Peripheral Neurovascular Dysfunction Related to B) Teaching the patient and family strict aseptic technique
Catheter Placement C) Teaching the patient and family how to set up the infusion
B) Ineffective Role Performance Related to Parenteral D) Teaching the patient to flush the line with sterile water
Nutrition E) Teaching the patient when it is safe to leave the access site
C) Bowel Incontinence Related to Parenteral Nutrition open to air
D) Chronic Pain Related to Catheter Placement
39. The nurse is caring for a patient who is postoperative from
34. A nurse is aware of the high incidence of catheter-related having a gastrostomy tube placed. What should the nurse do
bloodstream infections in patients receiving on a daily basis to prevent skin breakdown?
parenteral nutrition. What nursing action has the greatest A) Verify tube placement.
potential to reduce catheter-related bloodstream B) Loop adhesive tape around the tube and connect it securely
infections? to the abdomen.
A) Use clean technique and wear a mask during dressing C) Gently rotate the tube.
changes. D) Change the wet-to-dry dressing.
B) Change the dressing no more than weekly.
C) Apply antibiotic ointment around the site with each 40. A nurse is preparing to administer a patients scheduled
dressing change. parenteral nutrition (PN). Upon inspecting the
D) Irrigate the insertion site with sterile water during each bag, the nurse notices that the presence of small amounts of
dressing change. white precipitate are present in the bag What is the nurses
best action?
35. A patient who suffered a stroke had an NG tube inserted A) Recognize this as an expected finding.
to facilitate feeding shortly after admission. The B) Place the bag in a warm environment for 30 minutes.
patient has since become comatose and the patients family C) Shake the bag vigorously for 10 to 20 seconds.
asks the nurse why the physician is recommending the D)Contact the pharmacy to obtain a new bag of PN
removal of the patients NG tube and the insertion of a
gastrostomy tube. What is the nurses best response? Chapter 45: Management of Patients with Oral and
A) It eliminates the risk for infection. Esophageal Disorders
B) Feeds can be infused at a faster rate. 1. A nurse is providing oral care to a patient who is comatose.
C) Regurgitation and aspiration are less likely. What action best addresses the patients risk
D) It allows caregivers to provide personal hygiene more of tooth decay and plaque accumulation?
easily. A) Irrigating the mouth using a syringe filled with a
bacteriocidal mouthwash
36. A patient has been discharged home on parenteral B) Applying a water-soluble gel to the teeth and gums
nutrition (PN). Much of the nurses discharge education C) Wiping the teeth and gums clean with a gauze pad
focused on coping. What must a patient on PN likely learn to D) Brushing the patients teeth with a toothbrush and small
cope with? Select all that apply. amount of toothpaste
A) Changes in lifestyle
B) Loss of eating as a social behavior
C) Chronic bowel incontinence from GI changes
D) Sleep disturbances related to frequent urination during
nighttime infusions
E) Stress of choosing the correct PN formulation
2. An elderly patient comes into the emergency department
complaining of an earache. The patient and has an oral
37. A patient has a gastrostomy tube that has been placed to temperature of 100.2F and otoscopic assessment of the ear
drain stomach contents by low intermittent reveals a pearly gray tympanic membrane with no evidence of
suction. What is the nurses priority during this aspect of the discharge or inflammation. Which action should the triage
patients care? nurse take next?
A) Measure and record drainage. A) Palpate the patients parotid glands to detect swelling and
B) Monitor drainage for change in color. tenderness.
C) Titrate the suction every hour. B) Assess the temporomandibular joint for evidence of a
D) Feed the patient via the G tube as ordered. malocclusion.
C) Test the integrity of cranial nerve XII by asking the patient
38. A nurse is preparing to discharge a patient home on to protrude the tongue.
parenteral nutrition. What should an effective home D) Inspect the patients gums for bleeding and
care teaching program address? Select all that apply. hyperpigmentation.
A) Preparing the patient to troubleshoot for problems
8
3. A patient who had a hemiglossectomy earlier in the day is collaborated with the home health nurse to develop a plan of
assessed postoperatively, revealing a patent airway, stable care for this patient. What is a priority
vital signs, and no bleeding or drainage from the operative psychosocial outcome for a patient who has had a radical neck
site. The nurse notes the patient is alert. What is the patients dissection?
priority need at this time? A) Indicates acceptance of altered appearance and
A) Emotional support from visitors and staff demonstrates positive self-image
B) An effective means of communicating with the nurse B) Freely expresses needs and concerns related to
C) Referral to a speech therapist postoperative pain management
D) Dietary teaching focused on consistency of food and C) Compensates effectively for alteration in ability to
frequency of feedings communicate related to dysarthria
D) Demonstrates effective stress management techniques to
4. The nurse notes that a patient who has undergone skin, promote muscle relaxation
tissue, and muscle grafting following a modified radical neck
dissection requires suctioning. What is the most important 9. A patient has been diagnosed with an esophageal
consideration for the nurse when diverticulum after undergoing diagnostic imaging.
suctioning this patient? When taking the health history, the nurse should expect the
A) Avoid applying suction on or near the suture line. patient to describe what sign or symptom?
B) Position patient on the non operative side with the head of A) Burning pain on swallowing
the bed down. B) Regurgitation of undigested food
C) Assess the patients ability to perform self-suctioning. C) Symptoms mimicking a heart attack
D) Evaluate the patients ability to swallow saliva and clear D) Chronic parotid abscesses
fluids.
10. A nurse is caring for a patient who is acutely ill and has
5. A patient with gastroesophageal reflux disease (GERD) has a included vigilant oral care in the patients plan of
diagnosis of Barretts esophagus with minor cell changes. care. Why are patients who are ill at increased risk for
Which of the following principles should be integrated into developing dental caries?
the patients subsequent care? A) Hormonal changes brought on by the stress response cause
A) The patient will require an upper endoscopy every 6 an acidic oral environment
months to detect malignant changes. B) Systemic infections frequently migrate to the teeth
B) Liver enzymes must be checked regularly, as H2 receptor C) Hydration that is received intravenously lacks fluoride
antagonists may cause hepatic damage. D) Inadequate nutrition and decreased saliva production can
C) Small amounts of blood are likely to be present in the cause cavities
stools and are not cause for concern.
D) Antacids may be discontinued when symptoms of 11. A nurse who provides care in an ambulatory clinic
heartburn subside. integrates basic cancer screening into admission
assessments. What patient most likely faces the highest
6. The school nurse is planning a health fair for a group of fifth immediate risk of oral cancer?
graders and dental health is one topic that the nurse plans to A) A 65-year-old man with alcoholism who smokes
address. What would be most likely to increase the risk of B) A 45-year-old woman who has type 1 diabetes and who
tooth decay? wears dentures
A) Organic fruit juice C) A 32-year-old man who is obese and uses smokeless
B) Roasted nuts tobacco
C) Red meat that is high in fat D) A 57-year-old man with GERD and dental caries
D) Cheddar cheese 12. A nurse is caring for a patient who has undergone neck
resection with a radial forearm free flap. The nurses most
7. The nurses comprehensive assessment of a patient includes recent assessment of the graft reveals that it has a bluish color
inspection for signs of oral cancer. What and that mottling is visible. What is the nurses most
assessment finding is most characteristic of oral cancer in its appropriate action?
early stages? A) Document the findings as being consistent with a viable
A) Dull pain radiating to the ears and teeth graft.
B) Presence of a painless sore with raised edges B) Promptly report these indications of venous congestion.
C) Areas of tenderness that make chewing difficult C) Closely monitor the patient and reassess in 30 minutes.
D) Diffuse inflammation of the buccal mucosa D) Reposition the patient to promote peripheral circulation.

8. A medical nurse who is caring for a patient being discharged 13. A nurse is assessing a patient who has just been admitted
home after a radical neck dissection has to the postsurgical unit following surgical

9
resection for the treatment of oropharyngeal cancer. What 19. An emergency department nurse is admitting a 3-year-old
assessment should the nurse prioritize? brought in after swallowing a piece from a
A) Assess ability to clear oral secretions. wooden puzzle. The nurse should anticipate the
B) Assess for signs of infection. administration of what medication in order to relax the
C) Assess for a patent airway. esophagus to facilitate removal of the foreign body?
D) Assess for ability to communicate. A) Haloperidol
B) Prostigmine
14. A patient has been diagnosed with achalasia based on his C) Epinephrine
history and diagnostic imaging results. The nurse should D) Glucagon
identify what risk diagnosis when planning the patients care?
A) Risk for Aspiration Related to Inhalation of Gastric Contents 20. A nurse in an oral surgery practice is working with a
B) Risk for Imbalanced Nutrition: Less than Body patient scheduled for removal of an abscessed tooth. When
Requirements Related to Impaired Absorption providing discharge education, the nurse should recommend
C) Risk for Decreased Cardiac Output Related to Vasovagal which of the following actions?
Response A) Rinse the mouth with alcohol before bedtime for the next 7
D) Risk for Impaired Verbal Communication Related to Oral days.
Trauma B) Use warm saline to rinse the mouth as needed.
C) Brush around the area with a firm toothbrush to prevent
15. A nurse is providing health promotion education to a infection.
patient diagnosed with an esophageal reflux disorder. What D) Use a toothpick to dislodge any debris that gets lodged in
practice should the nurse encourage the patient to the socket.
implement?
A) Keep the head of the bed lowered. 21. A patient has been diagnosed with a malignancy of the
B) Drinka cup of hot tea before bedtime. oral cavity and is undergoing oncologic treatment. The
C) Avoid carbonated drinks. oncologic nurse is aware that the prognosis for recovery from
D) Eat a low-protein diet. head and neck cancers is often poor because of what
characteristic of these malignancies?
16. A staff educator is reviewing the causes of A) Radiation therapy often results in secondary brain tumors.
gastroesophageal reflux disease (GERD) with new staff B) Surgical complications are exceedingly common.
nurses. What area of the GI tract should the educator identify C) Diagnosis rarely occurs until the cancer is endstage.
as the cause of reduced pressure associated D) Metastases are common and respond poorly to treatment.
with GERD?
A) Pyloric sphincter 22. A patient has undergone surgery for oral cancer and has
B) Lower esophageal sphincter just been extubated in postanesthetic recovery.
C) Hypopharyngeal sphincter What nursing action best promotes comfort and facilitates
D) Upper esophageal sphincter spontaneous breathing for this patient?
A) Placing the patient in a left lateral position
17. A patient who has had a radical neck dissection is being B) Administering opioids as ordered
prepared for discharge. The discharge plan C) Placing the patient in Fowlers position
includes referral to an outpatient rehabilitation center for D) Teaching the patient to use the patient-controlled
physical therapy. What would the goals of analgesia (PCA) system
physical therapy for this patient include?
A) Muscle training to relieve dysphagia 23. A nurse is performing health education with a patient who
B) Relieving nerve paralysis in the cervical plexus has a history of frequent, serious dental caries. When
C) Promoting maximum shoulder function planning educational interventions, the nurse should identify
D) Alleviating achalasia by decreasing esophageal peristalsis a risk for what nursing diagnosis?
A) Ineffective Tissue Perfusion
B) Impaired Skin Integrity
18. A nurse is addressing the prevention of esophageal cancer C) Aspiration
in response to a question posed by a participant in a health D) Imbalanced Nutrition: Less Than Body Requirements
promotion workshop. What action has the greatest potential
to prevent esophageal cancer? 24. A patient has undergone rigid fixation for the correction of
A) Promotion of a nutrient-dense, low-fat diet a mandibular fracture suffered in a fight.
B) Annual screening endoscopy for patients over 50 with a What area of care should the nurse prioritize when planning
family history of esophageal cancer this patients discharge education?
C) Early diagnosis and treatment of gastroesophageal reflux A) Resumption of activities of daily living
disease B) Pain control
10
C) Promotion of adequate nutrition 30. A patients neck dissection surgery resulted in damage to
the patients superior laryngeal nerve. What area
25. A radial graft is planned in the treatment of a patients of assessment should the nurse consequently prioritize?
oropharyngeal cancer. In order to ensure that the surgery will A) The patients swallowing ability
be successful, the care team must perform what assessment B) The patients ability to speak
prior to surgery? C) The patients management of secretions
A) Assessing function of cranial nerves V, VI, and IX D) The patients airway patency
B) Assessing for a history of GERD
C) Assessing for signs or symptoms of atherosclerosis 31. A patient who underwent surgery for esophageal cancer is
D) Assessing the patency of the ulnar artery admitted to the critical care unit following
postanesthetic recovery. Which of the following should be
26. A nurse is caring for a patient who is postoperative day 1 included in the patients immediate postoperative plan of
following neck dissection surgery. The nurse is performing an care?
assessment of the patient and notes the presence of high- A) Teaching the patient to self-suction
pitched adventitious sounds over the patients trachea on B) Performing chest physiotherapy to promote oxygenation
auscultation. The patients oxygen saturation is 90% by pulse C) Positioning the patient to prevent gastric reflux
oximetry with a respiratory rate of 31 breaths per minute. D) Providing a regular diet as tolerated
What is the nurses most appropriate action?
A) Encourage the patient to perform deep breathing and 32. A patient has received treatment for oral cancer. The
coughing exercises hourly. combination of medications and radiotherapy has
B) Reposition the patient into a prone or semi-Fowlers resulted in leukopenia. Which of the following is an
position and apply supplementary oxygen by nasal cannula. appropriate response to this change in health status?
C) Activate the emergency response system. A) Ensure that none of the patients visitors has an infection.
D) Report this finding promptly to the physician and remain B) Arrange for a diet that is high in protein and low in fat.
with the patient. C) Administer colony stimulating factors (CSFs) as ordered.
D) Prepare to administer chemotherapeutics as ordered.
27. A nurse is caring for a patient who has just had a rigid
fixation of a mandibular fracture. When planning the 33. A nurse is caring for a patient who has had surgery for oral
discharge teaching for this patient, what would the nurse be cancer. When addressing the patients longterm needs, the
sure to include? nurse should prioritize interventions and referrals with what
A) Increasing calcium intake to promote bone healing goal?
B) Avoiding chewing food for the specified number of weeks A) Enhancement of verbal communication
after surgery B) Enhancement of immune function
C) Techniques for managing parenteral nutrition in the home C) Maintenance of adequate social support
setting D) Maintenance of fluid balance
D) Techniques for managing a gastrostomy
34. A patient with cancer of the tongue has had a radical neck
28. A community health nurse serves a diverse population. dissection. What nursing assessment would be
What individual would likely face the highest risk for parotitis? a priority for this patient?
A) A patient who is receiving intravenous antibiotic therapy in A) Presence of acute pain and anxiety
the home setting B) Tissue integrity and color of the operative site
B) A patient who has a chronic venous ulcer C) Respiratory status and airway clearance
C) An older adult whose medication regimen includes an 35. A patient returns to the unit after a neck dissection. The
anticholinergic surgeon placed a Jackson Pratt drain in the wound. When
D) A patient with poorly controlled diabetes who receives assessing the wound drainage over the first 24 postoperative
weekly wound care hours the nurse would notify the physician immediately for
what?
29. A nurse is providing care for a patient whose neck A) Presence of small blood clots in the drainage
dissection surgery involved the use of a graft. When B) 60 mL of milky or cloudy drainage
assessing the graft, the nurse should prioritize data related to C) Spots of drainage on the dressings surrounding the drain
what nursing diagnosis?
A) Risk for Disuse Syndrome 36. A nurse is caring for a patient who is postoperative from a
B) Unilateral Neglect neck dissection. What would be the most
C) Risk for Trauma appropriate nursing action to enhance the patients appetite?
D) Ineffective Tissue Perfusion A) Encourage the family to bring in the patients favored foods.
B) Limit visitors at mealtimes so that the patient is not
distracted.
11
C) Avoid offering food unless the patient initiates. 2. A patient comes to the clinic complaining of pain in the
D) Provide thorough oral care immediately after the patient epigastric region. What assessment question
eats. during the health interview would most help the nurse
determine if the patient has a peptic ulcer?
37. A patient with GERD has undergone diagnostic testing and A) Does your pain resolve when you have something to eat?
it has been determined that increasing the pace of gastric B) Do over-the-counter pain medications help your pain?
emptying may help alleviate symptoms. The nurse should C) Does your pain get worse if you get up and do some
anticipate that the patient may be prescribed what drug? exercise?
A) Metoclopramide (Reglan) D) Do you find that your pain is worse when you need to have
B) Omeprazole (Prilosec) a bowel movement?
C) Lansoprazole (Prevacid)
D) Famotidine (Pepcid) 3. A patient with a diagnosis of peptic ulcer disease has just
been prescribed omeprazole (Prilosec). How should the nurse
38. Results of a patient barium swallow suggest that the best describe this medications therapeutic action?
patient has GERD. The nurse is planning health education to A) This medication will reduce the amount of acid secreted in
address the patients knowledge of this new diagnosis. Which your stomach.
of the following should the nurse encourage? B) This medication will make the lining of your stomach more
A) Eating several small meals daily rather than 3 larger meals resistant to damage.
B) Keeping the head of the bed slightly elevated C) This medication will specifically address the pain that
C) Drinking carbonated mineral water rather than soft drinks accompanies peptic ulcer disease.
D) Avoiding food or fluid intake after 6:00 p.m.
4. A nurse is admitting a patient diagnosed with late-stage
39. A nurse is caring for a patient in the late stages of gastric cancer. The patients family is distraught and angry that
esophageal cancer. The nurse should plan to prevent she was not diagnosed earlier in the course of her disease.
or address what characteristics of this stage of the disease? What factor contributes to the fact that gastric cancer is often
Select all that apply. detected at a later stage?
A) Perforation into the mediastinum A) Gastric cancer does not cause signs or symptoms until
B) Development of an esophageal lesion metastasis has occurred.
C) Erosion into the great vessels B) Adherence to screening recommendations for gastric
D) Painful swallowing cancer is exceptionally low.
E) Obstruction of the esophagus C) Early symptoms of gastric cancer are usually attributed to
constipation.
40. A patient seeking care because of recurrent heartburn and D) The early symptoms of gastric cancer are usually not
regurgitation is subsequently diagnosed with a alarming or highly unusual.
hiatal hernia. Which of the following should the nurse include
in health education?
A) Drinking beverages after your meal, rather than with your
meal, may bring some relief.
B) Its best to avoid dry foods, such as rice and chicken, 5. A nurse is preparing to discharge a patient after recovery
because theyre harder to swallow. from gastric surgery. What is an appropriate
C) Many patients obtain relief by taking over-the-counter discharge outcome for this patient?
antacids 30 minutes before eating. A) The patients bowel movements maintain a loose
D) Instead of eating three meals a day, try eating smaller consistency.
amounts more often. B) The patient is able to tolerate three large meals a day.
C) The patient maintains or gains weight.
Chapter 46: Management of Patients with Gastric and D) The patient consumes a diet high in calcium.
Duodenal Disorders
6. A nurse caring for a patient who has had bariatric surgery is
1. A nurse is caring for a patient who just has been diagnosed
developing a teaching plan in anticipation of
with a peptic ulcer. When teaching the
the patients discharge. Which of the following is essential to
patient about his new diagnosis, how should the nurse best
include?
describe a peptic ulcer?
A) Drink a minimum of 12 ounces of fluid with each meal.
A) Inflammation of the lining of the stomach
B) Eat several small meals daily spaced at equal intervals.
B) Erosion of the lining of the stomach or intestine
C) Choose foods that are high in simple carbohydrates.
C) Bleeding from the mucosa in the stomach
D) Sit upright when eating and for 30 minutes afterward.
D) Viral invasion of the stomach wall

12
7. A nurse is completing a health history on a patient whose
diagnosis is chronic gastritis. Which of the 13. A patient with a peptic ulcer disease has had
data should the nurse consider most significantly related to metronidazole (Flagyl) added to his current medication
the etiology of the patients health problem? regimen. What health education related to this medication
A) Consumes one or more protein drinks daily. should the nurse provide?
B) Takes over-the-counter antacids frequently throughout the A) Take the medication on an empty stomach.
day. B) Take up to one extra dose per day if stomach pain persists.
C) Smokes one pack of cigarettes daily. C) Take at bedtime to mitigate the effects of drowsiness.
D) Reports a history of social drinking on a weekly basis. D) Avoid drinking alcohol while taking the drug.

8. A nurse in the postanesthesia care unit admits a patient 14. A patient was treated in the emergency department and
following resection of a gastric tumor. Following immediate critical care unit after ingesting bleach. What
recovery, the patient should be placed in which position to possible complication of the resulting gastritis should the
facilitate patient comfort and gastric emptying? nurse recognize?
A) Fowlers A) Esophageal or pyloric obstruction related to scarring
B) Supine B) Uncontrolled proliferation of H. pylori
C) Left lateral C) Gastric hyperacidity related to excessive gastrin secretion
D) Left Sims D) Chronic referred pain in the lower abdomen

9. A community health nurse is preparing for an initial home 15. A patient who underwent gastric banding 3 days ago is
visit to a patient discharged following a total having her diet progressed on a daily basis. Following her
gastrectomy for treatment of gastric cancer. What would the latest meal, the patient complains of dizziness and
nurse anticipate that the plan of care is most likely to include? palpitations. Inspection reveals that the patient is diaphoretic.
A) Enteral feeding via gastrostomy tube (G tube) What is the nurses best action?
B) Gastrointestinal decompression by nasogastric tube A) Insert a nasogastric tube promptly.
C) Periodic assessment for esophageal distension B) Reposition the patient supine.
D) Monthly administration of injections of vitamin B12 C) Monitor the patient closely for further signs of dumping
syndrome.
10. A nurse is assessing a patient who has peptic ulcer disease. D) Assess the patient for signs and symptoms of aspiration.
The patient requests more information about
the typical causes of Helicobacter pylori infection. What would 16. A patient is one month postoperative following restrictive
it be appropriate for the nurse to instruct the patient? bariatric surgery. The patient tells the clinic
A) Most affected patients acquired the infection during nurse that he has been having trouble swallowing for the past
international travel. few days. What recommendation should the nurse make?
B) Infection typically occurs due to ingestion of contaminated A) Eating more slowly and chewing food more thoroughly
food and water.
C) Many people possess genetic factors causing a 17. A patient is receiving education about his upcoming
predisposition to H. pylori infection. Billroth I procedure (gastroduodenostomy). This patient
11. A patient who experienced an upper GI bleed due to should be informed that he may experience which of the
gastritis has had the bleeding controlled and the patients following adverse effects associated with this procedure?
condition is now stable. For the next several hours, the nurse A) Persistent feelings of hunger and thirst
caring for this patient should assess for what signs and B) Constipation or bowel incontinence
symptoms of recurrence? C) Diarrhea and feelings of fullness
A) Tachycardia, hypotension, and tachypnea
B) Tarry, foul-smelling stools 18. A patient has experienced symptoms of dumping
C) Diaphoresis and sudden onset of abdominal pain syndrome following bariatric surgery. To what
D) Sudden thirst, unrelieved by oral fluid administration physiologic phenomenon does the nurse attribute this
syndrome?
12. A patient presents to the walk-in clinic complaining of A) Irritation of the phrenic nerve due to diaphragmatic
vomiting and burning in her mid-epigastria. The nurse knows pressure
that in the process of confirming peptic ulcer disease, the B) Chronic malabsorption of iron and vitamins A and C
physician is likely to order a diagnostic test to detect the C) Reflux of bile into the distal esophagus
presence of what? D) A sudden release of peptides
A) Infection with Helicobacter pylori
B) Excessive stomach acid secretion 19. A patient comes to the bariatric clinic to obtain
C) An incompetent pyloric sphincter information about bariatric surgery. The nurse assesses
D) A metabolic acidbase imbalance
13
the obese patient knowing that in addition to meeting the D) Unexplained bowel incontinence and fatty stools
criterion of morbid obesity, a candidate for bariatric surgery
must also demonstrate what? 25. A patient is recovering in the hospital following
A) Knowledge of the causes of obesity and its associated risks gastrectomy. The nurse notes that the patient has
B) Adequate understanding of required lifestyle changes become increasingly difficult to engage and has had several
C) Positive body image and high self-esteem angry outbursts at various staff members in recent days. The
D) Insight into why past weight loss efforts failed nurses attempts at therapeutic dialogue have been rebuffed.
What is the nurses most appropriate action?
20. A nurse is providing patient education for a patient with A) Ask the patients primary care provider to liaise between
peptic ulcer disease secondary to chronic the nurse and the patient.
nonsteroidal anti-inflammatory drug (NSAID) use. The patient B) Delegate care of the patient to a colleague.
has recently been prescribed misoprostol C) Limit contact with the patient in order to provide privacy.
(Cytotec). What would the nurse be most accurate in D) Make appropriate referrals to services that provide
informing the patient about the drug? psychosocial support.
A) It reduces the stomachs volume of hydrochloric acid
B) It increases the speed of gastric emptying 26. A patient has been admitted to the hospital after
C) It protects the stomachs lining diagnostic imaging revealed the presence of a gastric
D) It increases lower esophageal sphincter pressure outlet obstruction (GOO). What is the nurses priority
intervention?
21. A nurse is providing anticipator guidance to a patient who A) Administration of antiemetics
is preparing for bariatric surgery. The nurse learns that the B) Insertion of an NG tube for decompression
patient is anxious about numerous aspects of the surgery. C) Infusion of hypotonic IV solution
What intervention is most appropriate to alleviate the D) Administration of proton pump inhibitors as ordered
patients anxiety?
A) Emphasize the fact that bariatric surgery has a low risk of 27. A patient with a history of peptic ulcer disease has
complications. presented to the emergency department (ED) in distress.
B) Encourage the patient to focus on the benefits of the What assessment finding would lead the ED nurse to suspect
surgery. that the patient has a perforated ulcer?
C) Facilitate the patients contact with a support group. A) The patient has abdominal bloating that developed rapidly.
D) Obtain an order for a PRN benzodiazepine. B) The patient has a rigid, boardlike abdomen that is tender.
C) The patient is experiencing intense lower right quadrant
22. A patient has just been diagnosed with acute gastritis after pain.
presenting in distress to the emergency department with 28. Diagnostic imaging and physical assessment have revealed
abdominal symptoms. What would be the nursing care most that a patient with peptic ulcer disease has
needed by the patient at this time? suffered a perforated ulcer. The nurse recognizes that
A) Teaching the patient about necessary nutritional emergency interventions must be performed as
modification soon as possible in order to prevent the development of what
B) Helping the patient weigh treatment options complication?
C) Teaching the patient about the etiology of gastritis A) Peritonitis
D) Providing the patient with physical and emotional support B) Gastritis
C) Gastroesophageal reflux
23. A nurse is providing care for a patient who is D) Acute pancreatitis
postoperative day 2 following gastric surgery. The nurses
assessment should be planned in light of the possibility of 29. A nurse is performing the admission assessment of a
what potential complications? Select all that apply. patient whose high body mass index (BMI) corresponds to
A) Malignant hyperthermia class III obesity. In order to ensure empathic and patient-
B) Atelectasis centered care, the nurse should
C) Pneumonia do which of the following?
D) Metabolic imbalances A) Examine ones own attitudes towards obesity in general and
E) Chronic gastritis the patient in particular.
B) Dialogue with the patient about the lifestyle and
24. A patient is undergoing diagnostic testing for a tumor of psychosocial factors that resulted in obesity.
the small intestine. What are the most likely symptoms that C) Describe ones own struggles with weight gain and weight
prompted the patient to first seek care? loss to the patient.
A) Hematemesis and persistent sensation of fullness D) Elicit the patients short-term and long-term goals for
B) Abdominal bloating and recurrent constipation weight loss.
C) Intermittent pain and bloody stool
14
30. A patient has been prescribed orlistat (Xenical) for the A) Eat small, frequent meals with high calorie and vitamin
treatment of obesity. When providing relevant health content.
education for this patient, the nurse should ensure the patient B) Eat frequent meals with an equal balance of fat,
is aware of what potential adverse effect of treatment? carbohydrates, and protein.
A) Bowel incontinence C) Eat frequent, low-fat meals with high protein content.
B) Flatus with oily discharge D) Try to maintain the pre-diagnosis pattern of eating.
C) Abdominal pain
D) Heat intolerance 36. A nurse is caring for a patient who has a diagnosis of GI
bleed. During shift assessment, the nurse finds the patient to
31. A patient who is obese has been unable to lose weight betachycardic and hypotensive, and the patient has an
successfully using lifestyle modifications and has episode of hematemesis while the nurse is in the room. In
mentioned the possibility of using weight-loss medications. addition to monitoring the patients vital signs and level of
What should the nurse teach the patient about pharmacologic conscious, what would be a priority nursing action for this
interventions for the treatment of obesity? patient?
A) Weight loss drugs have many side effects, and most doctors A) Place the patient in a prone position.
think theyll all be off the market in a few years. B) Provide the patient with ice water to slow any GI bleeding.
B) There used to be a lot of hope that medications would help C) Prepare for the insertion of an NG tube.
people lose weight, but its been shown to be mostly a placebo D) Notify the physician.
effect.
C) Medications can be helpful, but few people achieve and 37. A nurse is caring for a patient hospitalized with an
maintain their desired weight loss with medications alone. exacerbation of chronic gastritis. What health
promotion topic should the nurse emphasize?
32. A patient has been diagnosed with peptic ulcer disease A) Strategies for maintaining an alkaline gastric environment
and the nurse is reviewing his prescribed medication regimen B) Safe technique for self-suctioning
with him. What is currently the most commonly used drug C) Techniques for positioning correctly to promote gastric
regimen for peptic ulcers? healing
A) Bismuth salts, antivirals, and histamine-2 (H2) antagonists D) Strategies for avoiding irritating foods and beverages
B) H2 antagonists, antibiotics, and bicarbonate salts
C) Bicarbonate salts, antibiotics, and ZES
D) Antibiotics, proton pump inhibitors, and bismuth salts

33. A patient who is obese is exploring bariatric surgery 38. A patient with gastritis required hospital treatment for an
options and presented to a bariatric clinic for preliminary exacerbation of symptoms and receives a subsequent
investigation. The nurse interviews the patient, analyzing and diagnosis of pernicious anemia due to malabsorption. When
documenting the data. Which of the following nursing planning the patients continuing care in the home setting,
diagnoses may be a contraindication for bariatric surgery? what assessment question is most relevant?
A) Disturbed Body Image Related to Obesity A) Does anyone in your family have experience at giving
B) Deficient Knowledge Related to Risks and Expectations of injections?
Surgery B) Are you going to be anywhere with strong sunlight in the
C) Anxiety Related to Surgery next few months?
D) Chronic Low Self-Esteem Related to Obesity C) Are you aware of your blood type?
D) Do any of your family members have training in first aid?
34. A patient has recently received a diagnosis of gastric
cancer; the nurse is aware of the importance of assessing the 39. A nurse is presenting a class at a bariatric clinic about the
patients level of anxiety. Which of the following actions is different types of surgical procedures offered by the clinic.
most likely to accomplish this? When describing the implications of different types of
A) The nurse gauges the patients response to hypothetical surgeries, the nurse should address which of the following
outcomes. topics? Select all that apply.
B) The patient is encouraged to express fears openly. A) Specific lifestyle changes associated with each procedure
C) The nurse provides detailed and accurate information B) Implications of each procedure for eating habits
about the disease. C) Effects of different surgeries on bowel function
D) The nurse closely observes the patients body language. D) Effects of various bariatric surgeries on fertility
E) Effects of different surgeries on safety of future
35. A patient has received a diagnosis of gastric cancer and is immunizations
awaiting a surgical date. During the preoperative period, the
patient should adopt what dietary guidelines? 40. A patient has come to the clinic complaining of pain just
above her umbilicus. When assessing the patient, the nurse
15
notes Sister Mary Josephs nodules. The nurse should refer the D) Administration of a glycerin suppository and an oral
patient to the primary care provider to be assessed for what laxative
health problem?
A) A GI malignancy 6. A patient admitted with inflammatory bowel disease asks
B) Dumping syndrome the nurse for help with menu selections. What menu selection
C) Peptic ulcer disease is most likely the best choice for this patient?
D) Esophageal/gastric obstruction A) Spinach
B) Tofu
Chapter 47: Management of Patients With Intestinal and C) Multigrain bagel
Rectal Disorders D) Blueberries
1. A nurse is working with a patient who has chronic
constipation. What should be included in patient 7. A patient is admitted to the medical unit with a diagnosis of
teaching to promote normal bowel function? intestinal obstruction. When planning this patients care,
A) Use glycerin suppositories on a regular basis. which of the following nursing diagnoses should the nurse
B) Limit physical activity in order to promote bowel peristalsis. prioritize?
C) Consume high-residue, high-fiber foods. A) Ineffective Tissue Perfusion Related to Bowel Ischemia
B) Imbalanced Nutrition: Less Than Body Requirements
2. A nurse is preparing to provide care for a patient whose Related to Impaired Absorption
exacerbation of ulcerative colitis has required hospital C) Anxiety Related to Bowel Obstruction and Subsequent
admission. During an exacerbation of this health problem, the Hospitalization
nurse would anticipate that the patients stools will have what D) Impaired Skin Integrity Related to Bowel Obstruction
characteristics?
A) Watery with blood and mucus 8. A nurse is presenting an educational event to a local
B) Hard and black or tarry community group. When speaking about colorectal cancer,
C) Dry and streaked with blood what risk factor should the nurse cite?
D) Loose with visible fatty streaks A) High levels of alcohol consumption
B) History of bowel obstruction
3. A patient has had an ileostomy created for the treatment of C) History of diverticulitis
irritable bowel disease and the patient is now preparing for D) Longstanding psychosocial stress
discharge. What should the patient be taught about changing 9. A patients screening colonoscopy revealed the presence of
this device in the home setting? numerous polyps in the large bowel. What principle should
A) Apply antibiotic ointment as ordered after cleaning the guide the subsequent treatment of this patients health
stoma. problem?
B) Apply a skin barrier to the peristomal skin prior to applying A) Adherence to a high-fiber diet will help the polyps resolve.
the pouch. B) The patient should be assured that these are a normal, age-
C) Dispose of the clamp with each bag change. related physiologic change.
4. A patient admitted with acute diverticulitis has experienced C) The patients polyps constitute a risk factor for cancer.
a sudden increase in temperature and
complains of a sudden onset of exquisite abdominal 10. A nursing instructor is discussing hemorrhoids with the
tenderness. The nurses rapid assessment reveals that nursing class. Which patients would the nursing instructor
the patients abdomen is uncharacteristically rigid on identify as most likely to develop hemorrhoids?
palpation. What is the nurses best response? A) A 45-year-old teacher who stands for 6 hours per day
A) Administer a Fleet enema as ordered and remain with the B) A pregnant woman at 28 weeks gestation
patient. C) A 37-year-old construction worker who does heavy lifting
B) Contact the primary care provider promptly and report D) A 60-year-old professional who is under stress
these signs of perforation.
C) Position the patient supine and insert an NG tube. 11. A nurse is planning discharge teaching for a 21-year-old
D) Page the primary care provider and report that the patient patient with a new diagnosis of ulcerative colitis. When
may be obstructed. planning family assessment, the nurse should recognize that
which of the following factors will likely have the greatest
5. A 35-year-old male patient presents at the emergency impact on the patients coping after discharge?
department with symptoms of a small bowel obstruction. In A) The familys ability to take care of the patients special diet
collaboration with the primary care provider, what needs
intervention should the nurse prioritize? B) The familys ability to monitor the patients changing health
A) Insertion of a nasogastric tube status
B) Insertion of a central venous catheter C) The familys ability to provide emotional support
C) Administration of a mineral oil enema
16
D) The familys ability to manage the patients medication B) Foul-smelling diarrhea that contains fat
regimen C) Fever accompanied by a rigid, tender abdomen
D) Bloody bowel movements accompanied by fecal
12. An older adult who resides in an assisted living facility has incontinence
sought care from the nurse because of recurrent episodes of
constipation. Which of the following actions should the nurse 17. A nurse is caring for a patient admitted with symptoms of
first perform? an anorectal infection; cultures indicate that
A) Encourage the patient to take stool softener daily. the patient has a viral infection. The nurse should anticipate
B) Assess the patients food and fluid intake. the administration of what drug?
C) Assess the patients surgical history. A) Acyclovir (Zovirax)
D) Encourage the patient to take fiber supplements. B) Doxycycline (Vibramycin)
C) Penicillin (penicillin
13. A 16-year-old presents at the emergency department
complaining of right lower quadrant pain and is 18. A nurse caring for a patient with colorectal cancer is
subsequently diagnosed with appendicitis. When planning this preparing the patient for upcoming surgery. The nurse
patients nursing care, the nurse should prioritize what nursing administers cephalexin (Keflex) to the patient and explains
diagnosis? what rationale?
A) Imbalanced Nutrition: Less Than Body Requirements A) To treat any undiagnosed infections
Related to Decreased Oral Intake B) To reduce intestinal bacteria levels
B) Risk for Infection Related to Possible Rupture of Appendix C) To reduce bowel motility
C) Constipation Related to Decreased Bowel Motility and
Decreased Fluid Intake 19. A nurse is teaching a group of adults about screening and
D) Chronic Pain Related to Appendicitis prevention of colorectal cancer. The nurse should describe
which of the following as the most common sign of possible
14. A nurse is talking with a patient who is scheduled to have colon cancer?
a hemicolectomy with the creation of a colostomy. The A) Development of new hemorrhoids
patient admits to being anxious, and has many questions B) Abdominal bloating and flank pain
concerning the surgery, the care of a stoma, and necessary C) Unexplained weight gain
lifestyle changes. Which of the following nursing actions is D) Change in bowel habits
most appropriate? 20. A nurse caring for a patient with a newly created
A) Reassure the patient that the procedure is relatively low ileostomy assesses the patient and notes that the patient
risk and that patients are usually successful in adjusting to an has had not ostomy output for the past 12 hours. The patient
ostomy. also complains of worsening nausea. What
B) Provide the patient with educational materials that match is the nurses priority action?
the patients learning style. A) Facilitate a referral to the wound-ostomy-continence
C) Encourage the patient to write down these concerns and (WOC) nurse.
questions to bring forward to the surgeon. B) Report signs and symptoms of obstruction to the physician.
D) Maintain an open dialogue with the patient and facilitate a C) Encourage the patient to mobilize in order to enhance
referral to the wound-ostomy- motility.
continence (WOC) nurse.
21. A nurse is working with a patient who is learning to care
15. A nurse is caring for a patient with constipation whose for a continent ileostomy (Kock pouch). Following the initial
primary care provider has recommended senna (Senokot) for period of healing, the nurse is teaching the patient how to
the management of this condition. The nurse should provide independently empty the ileostomy. The nurse should teach
which of the following education points? the patient to do which of the following actions?
A) Limit your fluid intake temporarily so you dont get A) Aim to eventually empty the pouch every 90 minutes.
diarrhea. B) Avoid emptying the pouch until it is visibly full.
B) Avoid taking the drug on a long-term basis. C) Insert the catheter approximately 5 cm into the pouch.
C) Make sure to take a multivitamin with each dose. D) Aspirate the contents of the pouch using a 60 mL piston
D) Take this on an empty stomach to ensure maximum effect. syringe.

16. The nurse is caring for a patient who is undergoing 22. A nurse is providing care for a patient who has a diagnosis
diagnostic testing for suspected malabsorption. When taking of irritable bowel syndrome (IBS). When
this patients health history and performing the physical planning this patients care, the nurse should collaborate with
assessment, the nurse should recognize what finding as most the patient and prioritize what goal?
consistent with this diagnosis? A) Patient will accurately identify foods that trigger symptoms.
A) Recurrent constipation coupled with weight loss B) Patient will demonstrate appropriate care of his ileostomy.
17
C) Patient will demonstrate appropriate use of standard 28. A nurse is providing care for a patient whose recent
infection control precautions. colostomy has contributed to a nursing diagnosis of
D) Patient will adhere to recommended guidelines for mobility Disturbed Body Image Related to Colostomy. What
and activity. intervention best addresses this diagnosis?
A) Encourage the patient to conduct online research into
23. A patient has been experiencing disconcerting GI colostomies.
symptoms that have been worsening in severity. B) Engage the patient in the care of the ostomy to the extent
Following medical assessment, the patient has been that the patient is willing.
diagnosed with lactose intolerance. The nurse should C) Emphasize the fact that the colostomy was needed to
recognize an increased need for what form of health alleviate a much more serious health
promotion? problem.
A) Annual screening colonoscopies D) Emphasize the fact that the colostomy is temporary
B) Adherence to recommended immunization schedules measure and is not permanent.
C) Regular blood pressure monitoring
D) Frequent screening for osteoporosis 29. A nurse is caring for a patient who has been admitted to
the hospital with diverticulitis. Which of the following would
24. An older adult has a diagnosis of Alzheimers disease and be appropriate nursing diagnoses for this patient? Select all
has recently been experiencing fecal incontinence. However, that apply.
the nurse has observed no recent change in the character of A) Acute Pain Related to Increased Peristalsis and GI
the patients stools. What is the nurses most appropriate Inflammation
intervention? B) Activity Intolerance Related to Generalized Weakness
A) Keep a food diary to determine the foods that exacerbate C) Bowel Incontinence Related to Increased Intestinal
the patients symptoms. Peristalsis
B) Provide the patient with a bland, low-residue diet. D) Deficient Fluid Volume Related to Anorexia, Nausea, and
C) Toilet the patient on a frequent, scheduled basis. Diarrhea
E) Impaired Urinary Elimination Related to GI Pressure on the
25. An adult patient has been diagnosed with diverticular Bladder
disease after ongoing challenges with constipation.
The patient will be treated on an outpatient basis. What 30. The nurse is providing care for a patient whose
components of treatment should the nurse inflammatory bowel disease has necessitated hospital
anticipate? Select all that apply. treatment. Which of the following would most likely be
A) Anticholinergic medications included in the patients medication regimen?
B) Increased fiber intake A) Anticholinergic medications 30 minutes before a meal
C) Enemas on alternating days B) Antiemetics on a PRN basis
D) Reduced fat intake C) Vitamin B12 injections to prevent pernicious anemia
E) Fluid reduction D) Beta adrenergic blockers to reduce bowel motility

26. A patients health history is suggestive of inflammatory 31. A patients colorectal cancer has necessitated a
bowel disease. Which of the following would suggest Crohns hemicolectomy with the creation of a colostomy. In the 4
disease, rather that ulcerative colitis, as the cause of the days since the surgery, the patient has been unwilling to look
patients signs and symptoms? at the ostomy or participate in any aspects of ostomy care.
A) A pattern of distinct exacerbations and remissions What is the nurses most appropriate response to this
B) Severe diarrhea observation?
C) An absence of blood in stool A) Ensure that the patient knows that he or she will be
D) Involvement of the rectal mucosa responsible for care after discharge.
B) Reassure the patient that many people are fearful after the
27. During a patients scheduled home visit, an older adult creation of an ostomy.
patient has stated to the community health nurse that she has C) Acknowledge the patients reluctance and initiate discussion
been experiencing hemorrhoids of increasing severity in of the factors underlying it.
recent months. The nurse should D) Arrange for the patient to be seen by a social worker or
recommend which of the following? spiritual advisor.
A) Regular application of an OTC antibiotic ointment
B) Increased fluid and fiber intake 32. A nurse is caring for an older adult who has been
C) Daily use of OTC glycerin suppositories experiencing severeClostridium difficile-related diarrhea.
D) Use of an NSAID to reduce inflammation When reviewing the patients most recent laboratory tests, the
nurse should prioritize which of
the following?
18
A) White blood cell level A) Close monitoring of temperature
B) Creatinine level B) Frequent abdominal auscultation
C) Hemoglobin level C) Assessment of hemoglobin, hematocrit, and red blood cell
D) Potassium level levels
D) Palpation of peripheral pulses and leg girth
33. A nurse is assessing a patients stoma on postoperative day
3. The nurse notes that the stoma has a shiny 38. A teenage patient with a pilonidal cyst has been brought
appearance and a bright red color. How should the nurse best for care by her mother. The nurse who is
respond to this assessment finding? contributing to the patients care knows that treatment will be
A) Irrigate the ostomy to clear a possible obstruction. chosen based on what risk?
B) Contact the primary care provider to report this finding. A) Risk for infection
C) Document that the stoma appears healthy and well B) Risk for bowel incontinence
perfused. C) Risk for constipation
D) Document a nursing diagnosis of Impaired Skin Integrity. D) Risk for impaired tissue perfusion

34. A patient has been diagnosed with a small bowel 39. A nurse at an outpatient surgery center is caring for a
obstruction and has been admitted to the medical unit. patient who had a hemorrhoidectomy. What discharge
The nurses care should prioritize which of the following education topics should the nurse address with this patient?
outcomes? A) The appropriate use of antibiotics to prevent postoperative
A) Preventing infection infection
B) Maintaining skin and tissue integrity B) The correct procedure for taking a sitz bath
C) Preventing nausea and vomiting C) The need to eat a low-residue, low-fat diet for the next 2
D) Maintaining fluid and electrolyte balance weeks
D) The correct technique for keeping the perianal region clean
without the use of water

35. A patients large bowel obstruction has failed to resolve 40. Which of the following is the most plausible nursing
spontaneously and the patients worsening condition has diagnosis for a patient whose treatment for colon
warranted admission to the medical unit. Which of the cancer has necessitated a colonostomy?
following aspects of nursing care is most appropriate for this A) Risk for Unstable Blood Glucose Due to Changes in
patient? Digestion and Absorption
A) Administering bowel stimulants as ordered B) Unilateral Neglect Related to Decreased Physical Mobility
B) Administering bulk-forming laxatives as ordered C) Risk for Excess Fluid Volume Related to Dietary Changes
C) Performing deep palpation as ordered to promote and Changes In Absorption
peristalsis D) Ineffective Sexuality Patterns Related to Changes in Self-
D) Preparing the patient for surgical bowel resection Concept

CHAPTER 43

1. The nurse is performing an initial assessment of a patient


reporting increased stomach acid related to stress. The nurse
36. A patient has been experiencing occasional episodes of knows that the healthcare provider will want to consider the
constipation and has been unable to achieve influence of what neuroregulator?
consistent relief by increasing physical activity and improving a. Gastrin
his diet. What pharmacologic intervention b. Cholecystokinin
should the nurse recommend to the patient for ongoing use? c. Norepinephrine
A) Mineral oil enemas d. Secretin
B) Bisacodyl (Dulcolax)
C) Senna (Senokot) 2. The nurse is performing an assessment of a patient. During
D) Psyllium hydrophilic mucilloid (Metamucil) the assessment the patient informs the nurse of some recent
"stomach trouble. " What does the nurse recognize is the
37. A patient with a diagnosis of colon cancer is 2 days most common symptom of patients with GI dysfunction?
postoperative following bowel resection and anastomosis. The a. Diffuse pain
nurse has planned the patients care in the knowledge of b. Dyspepsia
potential complications. What assessment should the nurse c. Constipation
prioritize? d. Abdominal bloating
19
d. The healthcare provider will be able to determine if there is
3. The nurse is investigating a patient's report of pain in the a presence of bowel disease e. The patient must have bowel
duodenal area. Where should the nurse perform the cleansing prior to the procedure
assessment?
a. Epigastric area and consider possible radiation of pain to 10. A patient is scheduled for a fiberoptic colo-noscopy. What
the right subscapular region does the nurse tell the patient fiberoptic colonoscopy is most
b. Hypogastrium in the right or left lower quadrant frequently used to diagnose?
c. Left lower quadrant a. Bowel disease of unknown origin
d. Periumbilical area, followed by the right lower quadrant b. Cancer screening
c. Inflammatory bowel disease
4. A patient reports abdominal pain associated with d. Occult bleeding
indigestion. What is characteristic of this type of pain?
a. Described as crampy or burning 11. A patient is being prepared for esophageal manometry.
b. In the left lower quadrant The nurse should inform the patient to withhold what
c. Less severe after an intake of fatty foods medication for 48 hours prior to the procedure?
d. Relieved by the intake of coarse vegetables, which a. Amiodarone
stimulate peristalsis b. Calan
C. Aspirin
5. The nurse is collecting a stool specimen from a patient. d. Metoprolol
What characteristic of the stool indicates to the nurse that the
patient may have an upper GI bleed?
a. Clay colored
b. Greasy and foamy
c. Tarry and black
d. Threaded with mucus 12. The nurse is assisting the healthcare provider with a
6. The nurse is performing an abdominal assessment for a gastric acid stimulation test for a patient. What medication
patient in the hospital with reports of abdominal pain. What should the nurse prepare to administer subcutaneously to
part of the assessment should the nurse perform first? a. stimulate gastric secretions?
Percussion b. Palpation c. Auscultation a. Inspection a. Pentagastrin
b. Atropine
7. The nurse has been directed to position a patient for an c. Glycopyrrolate
examination of the abdomen. What position should the nurse d. Acetylcysteine
place the patient in for the examination?
a. Prone position with pillows positioned to alleviate pressure 13. The nurse is assisting the healthcare provider with a
on the abdomen colonoscopy for a patient with rectal bleeding. The nurse is
b. Semi-Fowler position with the left leg bent to minimize asked to administer glucagon during the procedure. Why is
pressure on the abdomen the nurse asked to administer this medication during the
c. Supine position with the knees flexed to relax the procedure?
abdominal muscles d. Reverse Trendelenburg position to a. The patient is probably hypoglycemic and requires the
facilitate the natural propulsion of intestinal contents glucagon
b. To relieve anxiety during the procedure for moderate
8. The nurse auscultates a patient's abdomen to assess bowel sedation
sounds, documenting five to six sounds heard in less than 30 c. To reduce air accumulation in the colon
seconds. How does the nurse document the character of the d. To relax colonic musculature and reduce spasm
bowel sounds? 14. A patient is in the outpatient recovery area after having a
a. Normal colonoscopy and informs the nurse of abdominal cramping.
b. Hypoactive What is the best response by the nurse?
c. Hyperactive a. "We may need to go back in and see what - is wrong. You
d. Borborygmi shouldn't have discomfort"
b. "I will call the physician and let him know. He may have put
9. The nurse is providing instructions to a patient scheduled too much air in your colon"
for a gastroscopy. What should the nurse be sure to include in c. "I will call the physician and see if I can give you pain
the instructions? (Select all that apply.) medication. Sometimes the pain can be caused by having a
a. The patient must fast for 8 hours before the examination biopsy"
b. The throat will be sprayed with a local anesthetic d. "The cramping is caused by the air insuf-flated in the colon
c. After gastroscopy, the patient cannot eat or drink until the during the procedure"
gag reflex returns (1 to 2 hours)
20
15. During a colonoscopy with moderate seda-tion, the b. Keep the vent lumen above the patient's waist to prevent
patient groans with obvious discomfort and begins bleeding gastric content reflux
from the rectum. The patient is diaphoretic and has an c. Irrigate only through the vent lumen
increase in abdominal girth from distention. What d. Tape the tube to the head of the bed to avoid dislodgement
complication of this procedure is the nurse aware may be
occurring? 6. The nurse is inserting a nasoenteric tube for a patient with
a. Infection a paralytic ileus. How long does the nurse anticipate the tube
B. Bowel perforation will be required? (Select all that apply.) a.) Until bowel sound
c. Colonic polyp is present (b. Until flatus is passed c. Until peristalsis is
d. Rectal fissure resumed d. Until the patient stops vomiting e. Until the tube
comes out on its own
CHAPTER 44
7. The nurse assesses a patient who recently had a
1. The healthcare provider prescribes a naso-enteric feeding nasoenteric intubation. Symptoms of oligu-ria, lethargy, and
tube with a tungsten-weighted tip. The nurse knows to obtain tachycardia in the patient would indicate to the nurse what
what kind of tube for insertion? common complication? a. A cardiac dysrhythmia b, Fluid
a. A Dobbhoff or EnteraFio tube volume deficit c. Mucous membrane irritation d. Pulmonary
b. A Levin tube complications
c. A Salem Sump tube
d. A Sengstaken-Blakemore tube
8. The nurse checks residual content before each intermittent
tube feeding. When should the patient be reassessed?
2. A nurse prepares a patient for insertion of a nasoenteric a. When the residual is about 50 mL
tube. What position should the nurse place the patient in to b. When the residual is between 50 and 80 mL
facilitate insertion? c. When the residual is about 100 mL
(a) In high-Fowler position d. When the residual is greater than 200 ml
b. Flat in bed
c. On their right side 9. The nurse is caring for a patient who has dumping
d. In' semi-Fowler position with their head turned to the left syndrome from high-carbohydrate foods being administered
over a period of less than 20 minutes. What is a nursing
3. The nurse is inserting a Levin tube for a patient for gastric measure to prevent or minimize the dumping syndrome?
decompression. The tube should be inserted to 6 to 10 cm a. Administer the feeding at a warm temperature to decrease
beyond what length? peristalsis
a. A length of 50 cm (20 in) b. Administer the feeding by bolus to prevent continuous
b. b. A point that equals the distance from the nose to the intestinal distention
xiphoid process c. Administer the feeding with about 100 mL of fluid to dilute
c. The distance measured from the tip of the nose to the the high-carbohydrate concentration
earlobe and from the earlobe to the xiphoid process d. Administer the feeding with the patient in semi-Fowler
d. The distance determined by measuring. from the tragus of position to decrease transit time influenced by gravity
the ear to the xiphoid process
10. The nurse is caring for a comatose patient and
4. The nurse inserts a nasogastric tube into the right nares of a administering gastrostomy feedings. What does the nurse
patient. When testing the tube aspirate for pH to confirm understand is the reason that gastrostomy feedings are
placement, What does the nurse anticipate the pH will be if preferred to nasogastric feedings in the comatose patient?
placement is in the lungs? a. Gastroesophageal sphincter is intact, lessening the
а. 1 possibility of regurgitation
b. 2 b. Digestive process occurs more rapidly because the feedings
C. 4 do not have to pass through the esophagus
d. 6 c. Feedings can be administered with the patient in the
recumbent position
5. The nurse is managing a gastric Salem) sump tube for a 11. A patient has had a gastrostomy tube inserted. What does
patient who has an intestinal obstruction and will be going to the nurse anticipate the initial fluid nourishment will be after
surgery. What interventions should be the nurse perform to the insertion of the gastrostomy tube?
make sure the tube is functioning properly? a. Distilled water
a. Maintain intermittent or continuous suction at a rate b. 10% glucose and tap water
greater than 120 mm Hg c. Milk
d. High-calorie liquids
21
a. Candidiasis
12. The nurse is caring for a patient who has a gastrostomy b. Gingivitis
tube feeding. Upon initiating her care, the nurse aspirates the c. Herpes simplex
gastrostomy tube for gastric residual volume (GRV) and d. Cancer of the oral mucosa
obtains 200 mL, of gastric contents. What is the priority action
by the nurse? 3. The nurse is caring for a patient after drainage of a
a. Discontinue the infusion dentoalveolar or periapical abscess. What should the care of
b. Place the patient in a Fowler position with the head of the the patient include in the postoperative phase? (Select all that
bed at 45 degrees apply.)
c. Remove the aspirated fluid and do not reinstill the gastric a Soft diet after 24 hours
aspirate b.Fluid restriction for the first 48 hours because the gums are
d. Dilute the gastric tube feeding solution with water and swollen and painful
continue the feeding c. External heat by pad or compress to hasten the resolution
of the inflammatory swelling
13. The nurse is inserting a sump tube in a patient with d. Warm saline mouthwashes every 2 hours while awake
Crohn's disease who is suspected of having a bowel e. Gargle with peroxide and saline every 4 hours
obstruction. What does the nurse understand is the benefit of
the gastric (Salem) sump tube in comparison to some of the 4. A patient reports an inflamed salivary gland below the right
other tubes? ear. The nurse documents probable inflammation of which
a. The tube is radiopaque gland?
b. The tube is shorter a. The buccal gland
c. The tube is less expensive b. The parotid gland
d. The tube can be connected to suction and others cannot c. The sublingual gland
14. The nurse is inserting a nasogastric tube for a patient with d. The submandibular gland
pancreatitis. What intervention can the nurse provide to allow
facilitation of the tube insertion? 5. An older adult patient who has been living at home alone is
a. Spray the oropharynx with an anesthetic spray diagnosed with parotitis. What causative bacteria does the
b. Have the patient maintain a backward tilt head position c. nurse suspect is the cause of the parotitis?
Allow the patient to sip water as the tube is being inserted a. Methicillin-resistant Streptococcus aureus (MRSA)
d. Have the patient eat a cracker as the tube is being inserted b. Pneumococcus
c. Staphylococcus aureus
15. The nurse is inserting a nasogastric tube and the patient d. Streptococcus viridans
begins coughing and is unable to speak. What does the nurse
suspect has occurred? 6. A nurse inspects the Stensen duct of the parotid gland to
a. The nurse has inserted a tube that is too large for the determine inflammation and possible obstruction. What area
patient in the oral cavity would the nurse assess?
b. The nurse has inadvertently inserted the tube into the a. Buccal mucosa next to the upper molars
trachea b. Dorsum of the tongue
c. This is a normal occurrence and the tube should be left in c. Roof of the mouth next to the incisors
place d. Posterior segment of the tongue near the uvula
d. The tube is most likely defective and should be immediately
removed 7. The nurse is obtaining a history on a patient who comes to
the clinic. What symptom described by the patient is one of
CHAPTER 45 the first symptoms associated with esophageal disease?
1. The nurse is performing an assessment for a patient who a. Dysphagia
presents to the clinic with a lip lesion. The lesion is erythemic, b. Malnutrition
is fissuring, and has white hyperkeratosis. What does the c. Pain
nurse suspect that these findings are characteristic of? d. Regurgitation of food
a. Actinic cheilitis
b. Human papillomavirus lesion 8. A patient tells the nurse that it feels like food is "sticking" in
c. Frey syndrome the lower portion of the esophagus. What motility disorder
d. Sialadenitis does the nurse suspect these symptoms indicate?
a. Achalasia
2. A patient is experiencing painful, inflamed, and swollen b. Diffuse spasm
gums, and when brushing the teeth, the gums bleed. What c. Gastroesophageal reflex disease d. Hiatal hernia
common disease of the oral tissue does the nurse understand
these symptoms indicate?
22
9. A patient is brought to the emergency department by a c. Gonorrhea
family member, who states that the patient "drank drain d. Herpes simplex
cleaner." What intervention does the nurse anticipate
providing to treat this patient? (Select all that apply.) 15. A patient has been taking a 10-day course of antibiotics for
a. Administering an irritant that will stimulate vomiting pneumonia and now reports white patches that look like milk
b. Aspirating secretions from the pharynx if respirations are curds in the mouth. What treatment will the nurse educate
affected the patient about?
c. Neutralizing the chemical a. Nystatin
d. Washing the esophagus with large volumes of water b. Cephalexin
e. Administering activated charcoal c. Fluocinolone acetonide oral base gel
d. Acyclovir
10. The nurse is caring for a patient with a radica Deck
dissection and observes serosanguineout secretions in the
wound suction unit during the first postoperative day. What
amount of drainage in the wound unit should the nurse
expect to observe?
a. Between 40 and 80 mL
b. Approximately 80 to 120 mL CHAPTER 46
c. Between 120 and 160 ml 1. A patient has been diagnosed with acute gastritis and asks
d. Greater than 160 ml the nurse what could have caused it. What is the best
response by the nurse? (Select all that apply.)
11. A patient describes a burning sensation in the esophagus, a. "It can be caused by ingestion of strong acids"
pain when swallowing, and frequent indigestion. What does b. "You may have ingested some irritating foods"
the nurse suspect that these clinical manifestations indicate? (c. 'Is it possible that you are overusing aspirin"
a. This is an indication that the patient has peptic ulcer disease d. "It is a hereditary disease"
b. This is an indication that the patient may have esophageal e. "It is probably your nerves"
'cancer
c. These symptoms indicate gastrosophageal reflux disease 2. The nurse is caring for a patient who has been diagnosed
d. The clinical manifestations indicate diverticulitis with gastritis. To promote fluid balance when treating
gastritis, the nurse determines what minimal daily intake of
12. A patient has been diagnosed with Zenker diverticulum. fluids is required?
What treatment does the nurse anticipate educating the a. 1.0 L
patient about? b. 1.5 L
a. A low-residue diet c. 2.0 L
b. Chemotherapeutic agents d. 2.5 L
c. Radiation therapy
d. Surgical removal of the diverticulum 3. The nurse is preparing to administer medications to a
patient with acute gastritis. The patient has a new prescription
13. A patient who has positive human immune deficiency for a hista-mine-2 receptor antagonist. Which medication will
virus (HIV) comes to the clinic reporting white patches with the nurse administer?
rough hairlike projections on the tongue. The nurse observes a. Omeprazole
the lesions on the lateral border of the tongue. What b. Famotidine
abnormality of the mouth does the nurse determine these c. Lansoprazole
lesions are? d. Bismuth salts
a. Aphthous stomatitis
b. Nicotine stomatitis 4. A patient comes to the clinic stating, "I think I have an
c. Erythroplakia ulcer." What is a characteristic associated with peptic ulcer
d. Hairy leukoplakia pain that the nurse should inquire about?
(Select all that apply.)
14. A patient comes to the clinic reporting a sore throat. a. Burning sensation localized in the back or mid-epigastrium
When assessing the patient, the nurse observes a reddened b. Feeling of emptiness that precedes meals from 1 to 3 hours
ulcerated lesion on the lip that the patient states has been c. Severe gnawing pain that increases in severity as the day
there for a couple of weeks but is painless. What should the progresses
nurse consult with the healthcare provider about regarding d. Pain that radiates to the shoulder or jaw
testing? e. Vomiting without associated nausea
a. HIV 5. The nurse is educating a patient about discharge
b. Syphilis medications. When should the nurse instruct the patient to
23
take the antacid medication? 11. A patient taking metronidazole for the treatment of H.
a. With the meal pylori states that the medication is causing nausea. What
b. 30 minutes before the meal suggestion can the nurse provide to the patient to alleviate
c. 1 to 3 hours after the meal this problem?
d. Immediately after the meal a. Discontinue the use of the medication
b. Tell the patient to ask the healthcare provider to prescribe
6. A patient is scheduled for a Billroth I procedure for ulcer another type of antibiotic
management. What does the nurse understand will occur c. Take the medication with meals to decrease the nausea
when this procedure is performed? d. Crush the medication and put it in applesauce
a. A partial gastrectomy is performed with anastomosis of the
stomach segment to the duodenum
b. A sectioned portion of the stomach is joined to the jejunum
c. The antral portion of the stomach is removed and a
vagotomy is performed 12. The nurse is educating a patient with peptic ulcer disease
d. The vagus nerve is cut and gastric drainage is established about the disease process. What decreases the secretion of
bicarbonate from the pancreas into the duodenum, resulting
7. The nurse is developing a plan of care for a patient with in increased acidity of the duodenum?'
peptic ulcer disease. What nursing interventions should be a. Smoking
included in the care plan? (Select all that apply.) b. Eating spicy foods
a. Making neurovascular checks every 4 hours c. Drinking carbonated beverages
b. Frequently monitoring hemoglobin and hematocrit levels d. Taking antacids
(c. Observing stools and vomitus for color, consistency, and
volume 13. The nurse is educating a patient with peptic ulcer about
d. Checking the blood pressure and pulse rate every 15 to 20 dietary modification. What should the nurse be sure to
minutes include when educating this patient?
e. Inserting an indwelling catheter for incontinence a. Avoid extremes of temperature in food and beverages
b. Increase the fiber content in the diet
8. The nurse is caring for a patient who is suspected to have c. Decrease the amount of fluid the patient is drinking
developed a peptic ulcer hem-orrhage. Which action would d. Increase the protein content in the diet
the nurse perform first?
a. Place the patient in a recumbent position with the legs 14. The nurse determines that a patient is at risk for the
elevated development of gastric cancer. What foods should the nurse
b. Prepare a peripheral and central line for intravenous encourage the patient to avoid? (Select all that apply.)
infusion a. Fruits
c. Assess vital signs b. Vegetables ) Smoked foods d Pickled foods
d. Call the physician b. A reaction to the medication given for the ulcer
c. Gastric penetration
9. A patient sustained second- and third-degree burns over d. Perforation of the peptic ulcer
30% of the body surface area approximately 72 hours ago. e. Whole grains
What type of ulcer should the nurse be alert for while caring
for this patient? 15. A patient is having bile reflux after gastric surgery and
a. Curling ulcer having the pylorus removed. What pharmacologic therapy
b. Peptic ulcer should the nurse educate the patient regarding?
c. Esophageal ulcer a. The administration of capecitabine
d. Meckel ulcer b. The administration of omeprazole
c. The administration of famotidine
10. A patient is in the hospital for the treatment of peptic a. The administration of cholestyramine
ulcer disease. The nurse finds the patient with a board-like
abdomen, vomiting, and reports of a sudden severe pain in CHAPTER 47
the abdomen. What does the nurse suspect these symptoms 1. The nurse is assessing a patient with appendi-citis. The
indicate? nurse is attempting to elicit a Rovs-ing sign. Where should the
a. The treatment for the peptic ulcer is ineffective nurse palpate for this indicator of acute appendicitis?
b. A reaction to the medication given for the ulcer a. Right lower quadrant
c. Gastric penetration b. Left lower quadrant
d. Perforation of the peptic ulcer c. Right upper quadrant
d. Left upper quadrant

24
2. 2. A patient is not having daily bowel move. ments and has illness?
begun taking a laxative for this problem. What should the a. Ultrasound
nurse educate the patient about regarding laxative use? b. Endoscopy with mucosal biopsy
a. When taking the laxatives, plenty of fluids should be taken c. Stool specimen for ova and parasites
as well d. Pancreatic function tests
b. The laxatives should be taken no more than three times a
week or laxative addic. tion will result
c. Laxatives should not be routinely taken due to destruction
of nerve endings in the colon
d. Laxatives should never be the first response for the
treatment of constipation; natural methods should be
employed first

3. A patient is admitted to the hospital after not having had a 8. A patient arrives in the emergency department reporting
bowel movement in several days. The nurse observes the right lower abdominal pain that began 4 hours ago and is
patient is having small liquid stools, a grossly distended abdo- getting worse.
men, and abdominal cramping. What com. plication can this The nurse assesses rebound tenderness at McBurney point.
patient develop related to this problem? What does this assessment data indicate to the nurse?
a. Appendicitis a. Crohn's disease
b. Rectal fissures b. Ulcerative colitis
c. Bowel perforation n c. Appendicitis
d. Diverticulitis d. Diverticulitis

4. The nurse is performing an abdominal assessment for a 9. The nurse is caring for a patient who has had an
patient with diarrhea and auscul-tates a loud rumbling sound appendectomy. What is the best position for the nurse to
in the left lower quadrant. What will the nurse document this maintain the patient in after the surgery?
sound as on the nurse's notes? a. Prone
a. Loud bowel sounds b. Sims' left lateral
b. Borborygmus c. High Fowler
c. Tenesmus d. Supine with head of bed elevated 15 degrees
d. Peristalsis
10. A patient is suspected to have diverticulosis without
5. The nurse is caring for an older adult patient experiencing symptoms of diverticulitis. What diagnostic test does the
fecal incontinence. When planning the care of this patient, nurse anticipate educating the patient about prior to
what should the nurse designate as a priority goal? scheduling?
a. Maintaining skin integrity a. Colonoscopy
b. Beginning a bowel program to establish continence b. Barium enema
c. Instituting a diet high in fiber and increase fluid intake c. Flexible sigmoidoscopy
d. Determining the need for surgical intervention to correct d. CT scan
the problem
11. The nurse is admitting a patient with a diagnosis of
6. A patient with irritable bowel syndrome has been having diverticulitis and assesses that the patient has a board-like
more frequent symptoms lately and is not sure what lifestyle abdomen, no bowel sounds, and reports severe abdominal
changes may have occurred. What suggestion can the nurse pain. What is the nurse's first action?
provide to identify a trigger for the symptoms? a. Start an IV with lactated Ringer solution
a. Document how much fluid is being taken to determine if b. Notify the healthcare provider
the patient is overhydrating c. Administer a retention enema
b. Discontinue the use of any medication presently being d. Administer an opioid analgesic
taken to determine if medication is a trigger
c. Begin an exercise regimen and biofeedback to determine if 12. The nurse is assigned to care for a patient 2 days after an
external stress is a trigger. appendectomy due to a ruptured appendix with resultant
d. Keep a 1- to 2-week symptom and food diary to identify peritonitis. The nurse has just assisted the patient with ambu-
food triggers lation to the bedside commode when the patient points to the
surgical site and informs the nurse that "something gave
7. The nurse is caring for a patient who has malabsorption way." What does the nurse suspect may have occurred?
syndrome with an undetermined cause. What procedure will a. A drain may have become dislodged
the nurse assist with that is the best diagnostic test for this b. Wound dehiscence has occurred
25
c. Infection has developed
d. The surgical wound has begun to bleed

13. A patient is having a diagnostic workup for reports of


frequent diarrhea, right lower abdominal pain, and weight
loss. The nurse is reviewing the results of the barium study
and notes the presence of "string sign." What does the nurse
recognize that this is significant of?
a. Crohn's disease
b. Ulcerative colitis
c. Irritable bowel syndrome
d. Diverticulitis
14. A patient is being seen in the clinic reporting painful
hemorrhoids. The nurse assesses the patient and observes the
hemorrhoids are prolapsed but able to be placed back in the
rectum manually. The nurse documents the hemorrhoids as
what degree?
a. First degree
b. Second degree
c. Third degree
d. Fourth degree

15. The nurse is irrigating a colostomy when the patient


says,"You will have to stop, I am cramping so badly." What is
the priority action by the nurse?
a. Inform the patient that it will only last a minute and
continue with the procedure
b. Clamp the tubing and give the patient a rest period
c. Stop the irrigation and remove the tube
d. Replace the fluid with cooler water since it is probably too
warm

26

You might also like